Saltar al contenido principal
LibreTexts Español

24.1: Introducción- El problema con las ecuaciones de Maxwell

  • Page ID
    129349
  • \( \newcommand{\vecs}[1]{\overset { \scriptstyle \rightharpoonup} {\mathbf{#1}} } \) \( \newcommand{\vecd}[1]{\overset{-\!-\!\rightharpoonup}{\vphantom{a}\smash {#1}}} \)\(\newcommand{\id}{\mathrm{id}}\) \( \newcommand{\Span}{\mathrm{span}}\) \( \newcommand{\kernel}{\mathrm{null}\,}\) \( \newcommand{\range}{\mathrm{range}\,}\) \( \newcommand{\RealPart}{\mathrm{Re}}\) \( \newcommand{\ImaginaryPart}{\mathrm{Im}}\) \( \newcommand{\Argument}{\mathrm{Arg}}\) \( \newcommand{\norm}[1]{\| #1 \|}\) \( \newcommand{\inner}[2]{\langle #1, #2 \rangle}\) \( \newcommand{\Span}{\mathrm{span}}\) \(\newcommand{\id}{\mathrm{id}}\) \( \newcommand{\Span}{\mathrm{span}}\) \( \newcommand{\kernel}{\mathrm{null}\,}\) \( \newcommand{\range}{\mathrm{range}\,}\) \( \newcommand{\RealPart}{\mathrm{Re}}\) \( \newcommand{\ImaginaryPart}{\mathrm{Im}}\) \( \newcommand{\Argument}{\mathrm{Arg}}\) \( \newcommand{\norm}[1]{\| #1 \|}\) \( \newcommand{\inner}[2]{\langle #1, #2 \rangle}\) \( \newcommand{\Span}{\mathrm{span}}\)\(\newcommand{\AA}{\unicode[.8,0]{x212B}}\)

    En el Capítulo 23, resumimos nuestro conocimiento del electromagnetismo utilizando las cuatro ecuaciones de Maxwell. Por lo que podemos decir, esta es la mejor descripción que tenemos de los fenómenos eléctricos y magnéticos clásicos (clásicos en el sentido de que las ecuaciones no describen el comportamiento de las partículas que son descritas por la Mecánica Cuántica). Una de las consecuencias de las ecuaciones de Maxwell es que describen la existencia de ondas electromagnéticas que se propagan con una velocidad\(c\), dada por:\[\begin{aligned} c = \frac{1}{\sqrt{\epsilon_0\mu_0}}\end{aligned}\] dónde\(\epsilon_0\) y\(\mu_0\) son la permitividad y permeabilidad del espacio libre, respectivamente. La pregunta obvia que hay que hacer sobre estas ondas electromagnéticas es: “¿En qué medio se propagan estas ondas?”. A finales del siglo XIX, se pensó que el Universo estaba bañado en una sustancia llamada “éter luminoso” (o simplemente “éter”), a través de la cual se propagan las ondas electromagnéticas. Entonces se pensó que la velocidad,\(c\), de estas olas era, naturalmente, medida con respecto al éter. Esto llevó a la idea de que existe un marco inercial especial de referencia en el Universo, correspondiente a ese marco de referencia en el que la luz viaja a una velocidad,\(c\). Este marco de referencia estaría en reposo relativo al éter.

    A finales de la década de 1880, Michelson y Morley desarrollaron un ingenioso experimento para medir la velocidad de la Tierra en relación con el éter. Si el éter existe, y la Tierra se mueve a través de él, entonces un haz de luz que viaja paralelo al movimiento de la Tierra debería viajar a una velocidad ligeramente diferente a la de un haz de luz que viaja en dirección perpendicular. No obstante, Michelson y Morley demostraron de manera concluyente que no era así. No hay movimiento detectable de la Tierra a través de un medio en el que se propaga la luz (una onda electromagnética). No hay éter. Este fue un descubrimiento muy desconcertante, con extrañas implicaciones para las ecuaciones de Maxwell.

    Demostremos, a través de un simple ejemplo, un “problema” con la teoría del electromagnetismo. Más bien, no es un tema, sino una implicación muy extraña. Considera dos cables infinitamente largos, separados por una distancia\(r\), cada uno con una carga uniforme por unidad de longitud\(\lambda\), como se ilustra en la Figura\(\PageIndex{1}\).

    clipboard_ebeef3c19f18cfc7f42adaa650df54bf1.png
    Figura\(\PageIndex{1}\): Dos cables cargados infinitamente largos ejercen una fuerza eléctrica repulsiva entre sí.

    Podemos calcular fácilmente la magnitud de la fuerza eléctrica repulsiva,\(\vec F_E\), ejercida por un cable cargado en una sección de longitud\(l\),, del otro cable. La magnitud del campo eléctrico a una distancia,\(r\), desde un cable infinitamente largo con carga por unidad de longitud,\(\lambda\), viene dada por:\[\begin{aligned} E = \frac{\lambda}{2\pi \epsilon_0r}\end{aligned}\] Una sección de longitud,\(l\), del otro cable lleva carga,\(q=l\lambda\), de manera que la fuerza sobre esa sección de alambre tiene una magnitud:\[\begin{aligned} F_E=qE=\lambda l \left( \frac{\lambda}{2\pi \epsilon_0r}\right) = \frac{\lambda^2 l}{2\pi \epsilon_0r}\end{aligned}\] Y la fuerza por unidad de longitud, en cualquiera de los cables, tiene una magnitud:\[\begin{aligned} \frac{F_E}{l}=\frac{\lambda^2}{2\pi \epsilon_0r}\end{aligned}\] Esta es la única fuerza que se ejerce sobre uno de los alambres, y así nos permitirá especificar completamente el movimiento de ese alambre (conocemos todas las fuerzas ejercidas sobre el alambre, así que puede usar la Segunda Ley de Newton para determinar su aceleración y describir su movimiento).

    Considere los mismos dos cables, cada uno llevando carga por unidad de longitud pero como se ve desde un marco de referencia que se mueve hacia abajo (paralelo a los cables), con una velocidad,\(v\). En este marco de referencia, los cables infinitos aún tienen una carga neta por unidad de longitud, pero también parecen tener una corriente que se mueve hacia arriba\(I\), ya que observamos cargas positivas que se mueven hacia arriba a través del espacio.

    clipboard_ec0e3a65c94dc2f759b070f6460cbb48f.png
    Figura\(\PageIndex{2}\): Dos cables cargados infinitamente largos vistos desde un marco de referencia descendente parecerán tener corrientes ascendentes que resultarán en una fuerza magnética atractiva entre los cables.

    En este nuevo marco de referencia, vemos dos cables con cargas sobre ellos, moviéndose hacia arriba con velocidad,\(v\). En un lapso de tiempo\(\Delta t\),, vemos una longitud de cable,\(\Delta x=v\Delta t\), pasar, con carga total,\(\Delta Q=\lambda' \Delta x\). Por razones que quedarán claras a continuación, utilizamos una densidad de carga diferente,\(\lambda'\), en el marco de referencia móvil, aunque esperamos que eso\(\lambda'=\lambda\). Esto corresponde a una corriente,\(I\), dada por:\[\begin{aligned} I=\frac{\Delta Q}{\Delta t}=\lambda'\frac{\Delta x}{\Delta t}=\lambda' v\end{aligned}\] Así, en el marco de referencia que va hacia abajo, vemos dos cables con corriente ascendente en ellos, y estos cables deben extraer una fuerza magnética atractiva entre sí, con magnitud (por unidad de longitud):\[\begin{aligned} \frac{F'_B}{l} = -\frac{\mu_0 I_1I_2}{2\pi r}=-\frac{\mu_0 I^2}{2\pi r}=-\frac{\mu_0 \lambda'^2 v^2}{2\pi r}\end{aligned}\] donde el prime (') en la fuerza indica que la fuerza se mide en este diferente marco inercial de referencia, y el signo menos indica que está en la dirección opuesta a la fuerza eléctrica repulsiva.

    En el marco de referencia que va hacia abajo, los cables siguen cargados, y aún deben ejercer una fuerza repulsiva, con magnitud (por unidad de longitud):\[\begin{aligned} \frac{F'_E}{l}=\frac{\lambda'^2}{2\pi \epsilon_0r}\end{aligned}\] donde, nuevamente, utilizamos primos ('), para denotar cantidades que se miden en el marco de referencia móvil.

    La descripción de cómo se moverán los cables no debe depender del marco de referencia que escojamos para modelar los cables (se moverán bajo las fuerzas ejercidas sobre ellos independientemente de si los estamos observando desde un punto fijo o móvil, y de hecho independientemente de si los observamos en absoluto!). Así, la fuerza neta (por unidad de longitud) ejercida sobre un alambre no puede depender de nuestro marco de referencia. La fuerza eléctrica repulsiva total,\(F_E\), calculada en el marco estacionario de referencia debe ser igual a la suma de la fuerza magnética\(F'_B\), y eléctrica,\(F'_E\), calculada en el marco móvil de referencia 1:\[\begin{aligned} \frac{F_E}{l}&=\frac{F'_E}{l}+\frac{F'_B}{l}\\ \frac{\lambda^2}{2\pi \epsilon_0r}&=\frac{\lambda'^2}{2\pi \epsilon_0r} -\frac{\mu_0 \lambda'^2 v^2}{2\pi r}\end{aligned}\] donde reconocimos que la carga por unidad de longitud\(\lambda'\),, debe ser diferente en el marco de referencia móvil, o lo anterior daría una ecuación inconsistente (las fuerzas eléctricas cancelarían y encontraríamos que la fuerza magnética es igual a cero). Así, la fuerza eléctrica repulsiva debe ser mayor como se observa en el marco de referencia móvil, o la fuerza neta sobre el alambre sería diferente cuando se evalúa en los dos marcos de referencia. Esta es una conclusión verdaderamente extraña, como veremos.

    Antes de continuar, expongamos claramente nuestras suposiciones al modelar la fuerza entre los dos cables cargados:

    1. La fuerza neta sobre el alambre, que nos permite describir su movimiento, no puede depender de nuestro marco de referencia. Esperamos que las leyes de la física sean aplicables desde cualquier marco inercial de referencia.
    2. Suponemos que las ecuaciones de Maxwell se mantienen en todos los marcos inerciales de referencia. En particular, suponemos que las constantes,\(\mu_0\) y\(\epsilon_0\), son las mismas en todos los marcos de referencia inerciales.

    El primer supuesto permite afirmar que la fuerza neta en los dos marcos de referencia debe ser la misma. El segundo supuesto implica que debemos cambiar la densidad de carga,\(\lambda'\), en el marco de referencia móvil, ya que las constantes deben permanecer las mismas, y esta es la única cantidad que puede conducir a una fuerza eléctrica diferente en el marco de referencia móvil (que se requiere si la fuerza neta es ser lo mismo, según nuestro primer supuesto). Determinemos la nueva densidad de carga\(\lambda'\), en términos de la densidad de carga que se mide en reposo. Comenzando con el requisito de que la fuerza neta sobre el alambre no debe depender del marco de referencia, encontramos:\[\begin{aligned} \frac{\lambda^2}{2\pi \epsilon_0r}&=\frac{\lambda'^2}{2\pi \epsilon_0r} -\frac{\mu_0 \lambda'^2 v^2}{2\pi r}\\ \lambda^2&=\lambda'^2-\epsilon_0\mu_0\lambda'^2 v^2\\ \lambda^2&=\lambda'^2(1-\epsilon_0\mu_0v^2)\\ \therefore \lambda'&=\lambda \frac{1}{\sqrt{\epsilon_0\mu_0-v^2}}\end{aligned}\] Finalmente, reconociendo que podemos usar la velocidad de la luz\(c\),, para reemplazar la combinación de constantes\(\epsilon_0\mu_0\),, encontramos:\[\begin{aligned} \lambda'&=\lambda \frac{1}{\sqrt{1-\frac{v^2}{c^2}}}\end{aligned}\] Así, la carga por unidad de longitud en el cable es mayor cuando se mide desde el marco de referencia móvil (el factor que se multiplica\(\lambda\),, es mayor que uno si\(v<c\)). Debería ser algo molesto para usted que el cargo por unidad de longitud dependa del marco de referencia en el que se mide, pero esta es la única forma de que nuestras dos suposiciones se mantengan.

    Hasta el momento, esto solo ha sido algo matemático para asegurar que “las cosas salgan bien”, es decir, que nuestra descripción del movimiento del cable no dependa de nuestro marco de referencia. No obstante, las consecuencias de lo que acabamos de derivar son profundas. Concluimos que la carga por unidad de longitud en un cable depende de nuestro marco de referencia.

    Imagina dibujar dos líneas en uno de los cables, e imagina que en realidad puedes ver las cargas en el cable (tal vez fluorescen o algo así). La carga por unidad de longitud en el cable\(\lambda\),, se encuentra contando el número de cargas entre las dos líneas y dividiendo eso por la distancia entre las dos líneas. Ahora, tanto un observador en reposo con el cable, como uno que se mueve en relación con el cable acordarán el número de cargas contenidas entre las dos líneas. Ambos contarán el mismo número. Por lo tanto, si el observador que se mueve en relación con el cable va a medir una mayor densidad de carga, ¡entonces la distancia entre las líneas debe ser menor para ese observador! Para el observador que se mueve en relación con el cable, el cable es en realidad más corto. No parece ser más corto, ¡ES más corto!

    Para resumir, al exigir que las leyes de la física sean las mismas en todos los marcos inerciales de referencia, y al exigir que la ecuación de Maxwell sea la misma en todos los marcos inerciales de referencia, concluimos que la carga por unidad de longitud que se mide en un alambre debe depender del marco de referencia en el que se mide. Dado que no puede ser el número de cargas en el cable lo que depende del marco de referencia, debe ser la longitud del cable la que depende del marco de referencia. Así, o aceptamos que las ecuaciones de Maxwell son incorrectas, o aceptamos que son correctas pero que implican que los objetos se encogen en longitud cuando se mueven (independientemente de si están involucrados cargas). Resulta que esta última elección proporciona una mejor descripción de la naturaleza (¡y una que no ha sido invalidada!).

    Como consecuencia adicional de aceptar estas implicaciones de las ecuaciones de Maxwell es que la definición de los campos eléctrico y magnético debe depender del marco de referencia. En el ejemplo de esta sección, vimos que lo que parece un campo eléctrico en el marco estacionario de referencia, puede aparecer como la combinación de un campo magnético y eléctrico en un marco de referencia móvil.

    Notas al pie

    1. Esta afirmación es generalmente cierta para la Relatividad Especial, porque la fuerza se ejerce en la dirección perpendicular a la del movimiento.


    This page titled 24.1: Introducción- El problema con las ecuaciones de Maxwell is shared under a CC BY-SA license and was authored, remixed, and/or curated by Howard Martin revised by Alan Ng.